\documentclass[a4paper,10pt]{article} \usepackage{myXsim} %\usepackage[inline]{enumitem} %\usepackage{tasks} \title{DS 4} \tribe{Terminale L-ES} \date{20 décembre 2019} \xsimsetup{ solution/print = false } \begin{document} \maketitle Le barème est donné à titre indicatif, il pourra être modifié. Une part importante de la note sera dédiée à la rédaction, aux explications et à l'utilisation des notations mathématiques. \begin{exercise}[subtitle={Gestion d'un parc de vélos}] Afin de conserver au fil des années un parc en bon état, un loueur de vélos se sépare chaque hiver de 20\,\% de son stock et achète ensuite $35$ nouveaux vélos. On modélise la situation par une suite $\left(u_n\right)$ où, pour tout entier naturel $n$,\: $u_n$ représente le nombre de vélos présents dans le stock de ce loueur au 1\up{er} juillet de l'année $(2018 + n)$. Au 1\up{er} juillet 2018, le loueur possède $150$ vélos, ainsi $u_0 = 150$. \medskip \begin{enumerate} \item \begin{enumerate} \item Déterminer le nombre de vélos dans le stock du loueur au 1\up{er} juillet 2019. \item Justifier que, pour tout entier naturel $n$, on a : $u_{n+1} = 0,8u_n + 35$. \end{enumerate} \item On a calculé les premiers termes de cette suite à l'aide d'un tableur. Une copie d'écran est donnée ci-dessous: \begin{center} \begin{tabularx}{0.8\linewidth}{|*{3}{>{\centering \arraybackslash}X|}}\hline &A &B\\ \hline 1 &rang $n$ &terme $u_n$\\ \hline 2 &0 &150 \\ \hline 3 &1 &155\\ \hline 4 &2 &159 \\ \hline 5 &3 &162,2\\ \hline \end{tabularx} \end{center} \begin{enumerate} \item Quelle formule peut-on saisir dans la cellule B3 pour obtenir, par copie vers le bas, les termes successifs de la suite $\left(u_n\right)$ ? \item Pour les termes de rang 36, 37, 38, 39 et 40, on obtient les résultats suivants (arrondis au millième) : \begin{center} \begin{tabularx}{0.8\linewidth}{|*{3}{>{\centering \arraybackslash}X|}}\hline 38& 36& 174,992 \\ \hline 39& 37& 174,994\\ \hline 40& 38& 174,995\\ \hline 41& 39& 174,996\\ \hline 42& 40& 174,997\\ \hline \end{tabularx} \end{center} Conjecturer la limite de la suite $\left(u_n\right)$. \end{enumerate} \item \emph{Dans cette question, on cherche à démontrer la conjecture émise à la question précédente}. Pour cela, on pose pour tout entier naturel $n$ : $v_n= u_n - 175$. \begin{enumerate} \item Démontrer que la suite $\left(v_n\right)$ est une suite géométrique dont on précisera la raison et le premier terme. \item En déduire que, pour tout entier naturel $n$, on a : $u_n = - 25 \times 0,8^n + 175$. \item Déterminer alors la limite de la suite $\left(u_n\right)$. \end{enumerate} \item On admet que la suite $\left(u_n\right)$ est croissante. On veut déterminer la plus petite valeur de $n$ tels que : $u_n \geqslant 170$. \begin{enumerate} \item Compléter l'algorithme en annexe pour trouver cette valeur. \item Exécuter cet algorithme pour trouver la valeur de $n$. Interpréter le résultat dans le contexte de l'exercice. \end{enumerate} \end{minipage} \end{enumerate} \end{exercise} \begin{solution} \begin{enumerate} \item \begin{enumerate} \item $u_1=150\times 0,8+35=155$. Au 1\up{er} juillet 2019, le loueur aura 155 vélos. \item Le terme $u_n$ correspond au nombre de vélos l'année $(2018+n)$, $u_{n+1}$ le nombre de vélos l'année suivante. D'une année à l'autre il vend 20 \% de son stock, il lui en reste donc 80 \% soit $0,8\times u_n$. Puis il ajoute 35 nouveaux vélos. Donc il aura l'année suivante $0,8\times u_n+35$. Ainsi $\forall n\in \N$, $u_{n+1}=0,8u_n+35$ \end{enumerate} \item \begin{enumerate} \item Dans la cellule $B3$, il faut saisir : $=0,8*\text{B}2+35$ \item Le tableau donnant les termes de la suite pour $n$ allant de 38 à 42 permet de conjecturer que\\ $\lim\limits_{n \rightarrow +\infty} u_n=175$ \end{enumerate} \item On pose $\forall n\in \N$, $v_n=u_n-175$ \begin{enumerate} \item $\forall n\in \N$, $v_{n+1}=u_{n+1}-175=0,8u_n+35-175=0,8u_n-140=0,8\left(u_n-\dfrac{140}{0,8}\strut\right)=0,8(u_n-175)=0,8v_n$\\ Donc la suite $(v_n)$ est la suite géométrique de raison $q=0,8$ et de premier terme $v_0=u_0-175=-25$ \item $\forall n\in \N$, $v_n=v_0\times q^n=-25\times 0,8^n$. De plus $\forall n\in \N$, $u_n=v_n+175=-25\times 0,8^n+175$. \item La suite géométrique $(v_n)$ a pour raison $q=0,8$. $q\in]-1\,;\,1[$ donc $\displaystyle\lim_{n \to + \infty} 0,8^n = 0$ et $\lim\limits_{n \rightarrow +\infty} v_n=0$. $\lim\limits_{n \rightarrow +\infty} u_n=\lim\limits_{n \rightarrow +\infty} v_n+175=0+175=175$. \end{enumerate} \item \begin{enumerate} \item ~ \begin{center} \begin{minipage}{0.5\linewidth} \begin{algorithm}[H] \SetAlgoLined $u \leftarrow 150$ \; $n \leftarrow 0$ \; \Tq{$n < 179$}{ $u \leftarrow u*0.9+35$ \; $n \leftarrow n+1$ \; } \end{algorithm} \end{minipage} \end{center} \item En tâtonnant avec la calculatrice, on obtient \[ u_7 = 169,76 \qquad \qquad u_8 = 170,8 \] Donc au bout de 8 années, soit le 1\up{er} juillet 2026, le loueur possèdera plus de 170 vélos dans son stock. \end{enumerate} \end{enumerate} \end{solution} \pagebreak \begin{exercise}[subtitle={Club de foot}] Un club de football est composé d'équipes adultes masculines, adultes féminines et d'équipes d'enfants. Chaque week-end, la présidente Claire assiste au match d'une seule des équipes du club et elle suit: \begin{list}{\textbullet}{} \item dans 10\,\% des cas, le match d'une équipe adulte féminine; \item dans 40\,\% des cas, le match d'une équipe adulte masculine; \item dans les autres cas, le match d'une équipe d'enfants. \end{list} Lorsqu'elle assiste au match d'une équipe masculine, la probabilité que celle-ci gagne est $0,6$. Lorsqu'elle assiste au match d'une équipe d'enfants, la probabilité que celle-ci gagne est $0,54$. La probabilité que Claire voie l'équipe de son club gagner est $0,58$. On choisit un week-end au hasard. On note les évènements suivants: \begin{list}{\textbullet}{} \item $F$: \og Claire assiste au match d'une équipe adulte féminine \fg{}; \item $M$: \og Claire assiste au match d'une équipe adulte masculine \fg{}; \item $E$: \og Claire assiste au match d'une équipe d'enfants \fg{}; \item $G$: \og l'équipe du club de Claire gagne le match \fg{}. \end{list} \smallskip \emph{Pour tous évènements $A$ et $B$, on note $\overline{A}$ l'évènement contraire de $A$, $p(A)$ la probabilité de $A$ et, si $B$ est de probabilité non nulle, $p_{B}(A)$ la probabilité de $A$ sachant $B$.} \medskip \begin{enumerate} \item L'arbre de probabilité est donné en \textbf{annexe}. Le compléter au fur et à mesure de l'exercice. \item Déterminer la probabilité $p(M \cap G)$. \item \begin{enumerate} \item Démontrer que $p(F \cap G) = 0,07$. \item En déduire $p_{F}(G)$. \item La probabilité que l'équipe adulte féminine gagne un match est $0,47$. La présence de Claire semble-t-elle favoriser la victoire de l'équipe adulte féminine? \end{enumerate} \item Claire annonce avoir assisté à la victoire d'une équipe du club. Quelle est la probabilité qu'elle ait suivi le match d'une équipe adulte féminine ? \end{enumerate} \end{exercise} \begin{solution} \begin{enumerate} \item ~ \begin{center} \begin{tikzpicture}[grow=right, xscale=2] \node {.} child {node {$E$} child {node {$G$} edge from parent node[above] {0.54} } child {node {$\overline{G}$} edge from parent node[above] {0.46} } edge from parent node[above] {0.5} } child[missing] {} child {node {$M$} child {node {$G$} edge from parent node[above] {0.6} } child {node {$\overline{G}$} edge from parent node[above] {0.4} } edge from parent node[above] {0.4} } child[missing] {} child { node {$F$} child {node {$G$} edge from parent node[above] {0.7} } child {node {$\overline{G}$} edge from parent node[above] {0.3} } edge from parent node[above] {0.1} } ; \end{tikzpicture} \end{center} \item $p(M \cap G)= p(M)\times p_{M}(G)=0,4 \times 0,6=0,24$ \item \begin{enumerate} \item D'après la formule des probabilités totales: $p(G)=p(F\cap G) + p(M\cap G)+p(E\cap G)$. On sait que $p(G)=0,58$ et que $p(M\cap G)=0,24$. \[ p(E \cap G)= p(E)\times p_{E}(G)=0,5 \times 0,54=0,27 \] On en déduit que $p(F \cap G) = p(G) - p(M\cap G) - p(E\cap G) = 0,58-0,24-0,27 = 0,07$. \item $p(F \cap G) = p(F)\times p_{F}(G)$ donc \[ p_{F}(G) = \dfrac{p(F \cap G)}{p(F)} = \dfrac{0,07}{0,1}=0,7 \] On peut ainsi compléter l'arbre (voir \textbf{annexe)}. \item La probabilité que l'équipe adulte féminine gagne un match est $0,47$.% La présence de Claire semble-t-elle favoriser la victoire de l'équipe adulte féminine? La probabilité que l'équipe féminine gagne un match sachant que Claire a assisté au match est $p_{F}(G) = 0,7$. Donc la présence de Claire semble favoriser la victoire de l'équipe féminine. \end{enumerate} \item Claire annonce avoir assisté à la victoire d'une équipe de club. La probabilité qu'elle ait suivi le match d'une équipe adulte féminine est $p_{G}(F)=\dfrac{p(F\cap G)}{p(G)} = \dfrac{0,07}{0,58} \approx 0,12$. \end{enumerate} \end{solution} \begin{exercise}[subtitle={Vrai ou Faux}] Pour chacune des affirmations suivantes, indiquer si elle est vraie ou fausse et justifier la réponse donnée. \begin{enumerate} \item On considère l'arbre pondéré suivant: \begin{center} \begin{tikzpicture}[xscale=2, grow=right] \node {.} child {node {$R$} child {node {$S$} edge from parent node[below] {0.4} } child {node {$\overline{S}$} edge from parent node[above] {0.6} } edge from parent node[below] {0.7} } child[missing] {} child { node {$\overline{R}$} child {node {$S$} edge from parent node[below] {0.2} } child {node {$\overline{S}$} edge from parent node[above] {0.8} } edge from parent node[above] {0.3} } ; \end{tikzpicture} \end{center} \textbf{Affirmation 1}: La probabilité de $\overline{R}$ sachant $S$ est $0,06$. \medskip \item Soit $k$ un réel tel que $0 \leqslant k < 18$. Soit $X$ une variable aléatoire qui suit la loi uniforme sur l'intervalle $\left [k~;\, 18\strut\right ]$. On suppose que l'espérance de $X$ est égale à 12. \smallskip \textbf{Affirmation 2}: La valeur de $k$ est 9. \medskip \item Soit $f$ une fonction dérivable sur l'intervalle $\left [0~;\, 15\strut\right ]$. On suppose que sa fonction dérivée, notée $f'$, est continue sur $\left [0~;\, 15\strut\right ]$. Les variations de $f'$ sont représentées dans le tableau ci-dessous. \begin{minipage}{0.4\linewidth} \begin{tikzpicture}[baseline=(a.north)] \tkzTabInit[lgt=1,espcl=2]{$x$/1,$f'(x)$/2}{0, 5, 15} \tkzTabVar{+/ 30, -/ -5, +/ 10} \end{tikzpicture} \end{minipage} \begin{minipage}{0.6\linewidth} \textbf{Affirmation 3}: La courbe représentative $\mathcal{C}_f$ de la fonction $f$ admet une et une seule tangente parallèle à l'axe des abscisses. \bigskip \textbf{Affirmation 4}: La fonction $f$ est convexe sur $\left [5~;\, 15\strut\right ]$. \end{minipage} \end{enumerate} \end{exercise} \begin{solution} \begin{enumerate} \item On a $P_S\left(\overline{R}\right) = \dfrac{P\left(S \cap \overline{R}\right)}{P(S)}$. $\bullet~~$ $P\left(S \cap \overline{R}\right) = P\left(\overline{R} \cap S\right) = P\left(\overline{R} \right) \times P_{\overline{R}}(S) = 0,3 \times 0,2 : 0,06$. $\bullet~~$ D'après la loi des probabilités totales : $P(S) = P(R \cap S) + P\left(\overline{R} \cap S \right) = P(R) \times P_R (S) + P\left(\overline{R} \right) \times P_{\overline{R}}(S) = 0,7 \times 0,4 + 0,3 \times 0,2 = 0,28 + 0,06 = 0,34$. Donc $\dfrac{P\left(S \cap \overline{R}\right)}{P(S)} = \dfrac{0,6}{0,34} \approx 0,18 \neq 0.06$ : l'affirmation est fausse. \medskip \item L'espérance de $X$sur $[k~;~18]$ est égale à $\dfrac{k + 18}{2} = 12 \iff k +18 = 24 \iff k = 6$ : l'affirmation est fausse. \medskip \item \smallskip \textbf{Affirmation 3}: La courbe représentative $\mathcal{C}_f$ de la fonction $f$ admet une et une seule tangente parallèle à l'axe des abscisses. \smallskip D'après le tableau de variations de $f'$, cette dérivée s'annule sur l'intervalle [0~;~5] et sur l'intervalle [5~;~15]. Il existe donc $a \in [0~;~5[$ tel que $f'(a) = 0$ et $b \in ]5~;~15]$ tel que $f'(b) = 0$. En ces deux points distincts le nombre dérivé est nul ce qui signifie que les tangentes à la courbe $\mathcal{C}_f$ sont horizontales : l'affirmation est fausse. \textbf{Affirmation 4}: La fonction $f$ est convexe sur $\left [5~;\, 15\strut\right ]$. D'après le tableau de variations $f'$croissante sur $[5~;~15]$ donc la fonction $f$ est convexe sur cet intervalle : affirmation vraie. \end{enumerate} \end{solution} \end{document} %%% Local Variables: %%% mode: latex %%% TeX-master: "master" %%% End: